LSAT and Law School Admissions Forum

Get expert LSAT preparation and law school admissions advice from PowerScore Test Preparation.

 Administrator
PowerScore Staff
  • PowerScore Staff
  • Posts: 8916
  • Joined: Feb 02, 2011
|
#24995
Complete Question Explanation

Strengthen. The correct answer choice is (A)

This stimulus contains an argument about sentencing guidelines. In some areas, the penalty for theft and the penalty for bribery is the same. The author concludes that lawmakers in those jurisdictions consider the harm resulting from theft to be the same as the harm resulting from bribery. The fact that the penalties for committing two crimes are identical does not necessarily allow any further conclusions to be drawn. Without more information, we do not know the rationale behind the sentencing guidelines, or even if there was a thoughtful plan behind the sentencing structure. The argument does not connect the penalties assessed against a crime, and the perceived harm caused by that crime. The correct answer choice should make that connection.

Answer choice (A): This is the correct answer choice. It connects the penalty assessed against a crime with the amount of harm lawmakers believe that crime will cause. If the penalties are proportional to the amount of harm believed to be caused by a crime, it strengthens the conclusion that the lawmakers believe that theft and bribery cause an equivalent amount of harm.

Answer choice (B): This argument is about comparing two acts that are already illegal, and the potential penalties for committing those acts. Information about what lawmakers’ decision process in determining whether an act should be illegal is not relevant to the penalty structure determined for acts that are already illegal.

Answer choice (C): In this stimulus, author agues about equivalent penalties, and not about increased penalties. Further, the stimulus does not discuss the process for changing penalties for crimes, but only about punishments that have already been set. The answer choice does not impact the argument.

Answer choice (D): The argument is about lawmakers’ perception of harm, rather than objective harm caused. Additionally, it is possible for bribery or theft to harm someone other than just the victim. Bribery, for example, harms not only the victim, but also society by undermining public support in the institution that was accepting bribes.

Answer choice (E): The conditional reasoning in this answer choice makes it incorrect. The argument does not say that the lawmakers’ goal is to deter crime. It also does not say that the penalties are necessarily proportional. The argument needs the additional fact, as in answer choice (B), that the penalties are meant to be proportional to the harm caused by the specific crime.
 akanshalsat
  • Posts: 104
  • Joined: Dec 20, 2017
|
#49382
Hello :)

So I kind of guessed on this, and even reading through the explanation I don't think It fully has clicked for me. How exactly is A strengthening it? you might have to dissect it a little more bc I don't think I'm fully grasping the connection.
 Vaidehi Joshi
PowerScore Staff
  • PowerScore Staff
  • Posts: 23
  • Joined: Aug 16, 2018
|
#53469
This is almost similar in its approach to an assumption question. ask yourself if the author makes a jump in logic/assumption. the author argues:

lawmakers make sentencing guidelines that mandate a penalty for theft identical to the one they have mandated for bribery
-->
THUS, lawmakers in those jurisdictions evidently consider the harm resulting from theft to be equal to the harm resulting from bribery.

what's missing? this should be your prephrase, even though it's not really an assumption question (it's still in the HELP family!). my prephrase was: "hey, sounds like they think that if two things should have equal punishment, then those two things cause similar amounts of harm/lawmakers seem to think harm should be tied to the punishment for a crime"
and this prephrase is indeed reflected in (A)
 LSAT2018
  • Posts: 242
  • Joined: Jan 10, 2018
|
#61512
Just to clarify, the author assumes a correlation between penalty and harm in the stimulus (this is unstated), and the answer strengthens this correlation by explicitly stating that they are proportional?
User avatar
 Dave Killoran
PowerScore Staff
  • PowerScore Staff
  • Posts: 5853
  • Joined: Mar 25, 2011
|
#61522
LSAT2018 wrote:Just to clarify, the author assumes a correlation between penalty and harm in the stimulus (this is unstated), and the answer strengthens this correlation by explicitly stating that they are proportional?
Correct!

Get the most out of your LSAT Prep Plus subscription.

Analyze and track your performance with our Testing and Analytics Package.